Find log_3 (a2+a3+a4+a5+a6+a7)

  • MHB
  • Thread starter Albert1
  • Start date
In summary, the given equation can be simplified to find the values of $a_2, a_3, ..., a_6, a_7$ which are all integers. From this, we can use the values to find the logarithm of the sum of these values which equals to 2. However, for a unique solution, an additional restriction of $0\leq a_i<i$ must be added.
  • #1
Albert1
1,221
0
$\dfrac {5}{7}=\dfrac {a_2}{2!}+\dfrac {a_3}{3!}+\dfrac {a_4}{4!}+\dfrac {a_5}{5!}+\dfrac {a_6}{6!}+\dfrac {a_7}{7!}$
here :$a_2,a_3,----,a_6,a_7\in Z$
pease find:$log_3 (a_2+a_3+a_4+a_5+a_6+a_7)=?$
 
Mathematics news on Phys.org
  • #2
Albert said:
$\dfrac {5}{7}=\dfrac {a_2}{2!}+\dfrac {a_3}{3!}+\dfrac {a_4}{4!}+\dfrac {a_5}{5!}+\dfrac {a_6}{6!}+\dfrac {a_7}{7!}$
here :$a_2,a_3,----,a_6,a_7\in Z$
pease find:$log_3 (a_2+a_3+a_4+a_5+a_6+a_7)=?$

My Solution:

Multiply both side by \(\displaystyle 7!\;,\) we get

\(\displaystyle \displaystyle 3600 = 2520a_{2}+840a_{3}+210a_{4}+42a_{5}+7a_{6}+a_{7}\)

Now \(\displaystyle 3600-a_{7}\) is a multiply of \(\displaystyle 7\). So \(\displaystyle a_{1}=2\)

Thus \(\displaystyle \displaystyle \frac{3598}{7} = 514=360a_{2}+120a_{3}+30a_{4}+6a_{5}+a_{6}\)

So \(\displaystyle 3598-a_{6}\) is a multiple of \(\displaystyle 6\;,\) So \(\displaystyle a_{6} = 4\)

Thus \(\displaystyle \displaystyle \frac{510}{6}=85=60a_{2}+20a_{3}+5a_{4}+a_{5}\)

Thus \(\displaystyle 85-a_{5}\) is a multiple of \(\displaystyle 5\;,\) where \(\displaystyle a_{5}=0\)

Continuing in this process, we get \(\displaystyle a_{4}=1\;a_{3}=1\;,a_{2}=1\)

So we get \(\displaystyle a_{2}+a_{3}+a_{4}+a_{5}+a_{6}+a_{7}=1+1+1+0+4+2=9\)

So \(\displaystyle \log_{3}\left(a_{2}+a_{3}+a_{4}+a_{5}+a_{6}+a_{7}\right)=\log_{3}(9)=2\)
 
Last edited by a moderator:
  • #3
jacks said:
[hide]My Solution:: Multiply both side by \(\displaystyle 7!\;,\) we get

\(\displaystyle \displaystyle 3600 = 2520a_{2}+840a_{3}+210a_{4}+42a_{5}+7a_{6}+a_{7}\)

Now \(\displaystyle 3600-a_{7}\) is a multiply of \(\displaystyle 7\). So \(\displaystyle a_{1}=2\)

Thus \(\displaystyle \displaystyle \frac{3598}{7} = 514=360a_{2}+120a_{3}+30a_{4}+6a_{5}+a_{6}\)

So \(\displaystyle 3598-a_{6}\) is a multiple of \(\displaystyle 6\;,\) So \(\displaystyle a_{6} = 4\)

Thus \(\displaystyle \displaystyle \frac{510}{6}=85=60a_{2}+20a_{3}+5a_{4}+a_{5}\)

Thus \(\displaystyle 85-a_{5}\) is a multiple of \(\displaystyle 5\;,\) where \(\displaystyle a_{5}=0\)

Continuing in this process, we get \(\displaystyle a_{4}=1\;a_{3}=1\;,a_{2}=1\)

So we get \(\displaystyle a_{2}+a_{3}+a_{4}+a_{5}+a_{6}+a_{7}=1+1+1+0+4+2=9\)

So \(\displaystyle \log_{3}\left(a_{2}+a_{3}+a_{4}+a_{5}+a_{6}+a_{7}\right)=\log_{3}(9)=2\)[/hide]

above is a good solution but

$a_7=9$ and $a_6=3$ is also a solution so some attribute of problem is missing
 
  • #4
kaliprasad said:
above is a good solution but

$a_7=9$ and $a_6=3$ is also a solution so some attribute of problem is missing
yes you are right
if we add a restriction:$0\leq a_i<i$
then the solution will be unique
 
  • #5


I would first clarify that the notation used in the provided content is not standard mathematical notation. However, I can still provide a response based on my understanding of the given information.

Based on the given equation, it appears that the values of $a_2, a_3, ..., a_6, a_7$ are all integers. Therefore, we can rewrite the equation as:

$\dfrac {5}{7}=\dfrac {a_2}{2!}+\dfrac {a_3}{3!}+\dfrac {a_4}{4!}+\dfrac {a_5}{5!}+\dfrac {a_6}{6!}+\dfrac {a_7}{7!}=\dfrac {a_2}{2}+\dfrac {a_3}{6}+\dfrac {a_4}{24}+\dfrac {a_5}{120}+\dfrac {a_6}{720}+\dfrac {a_7}{5040}$

Solving for $a_2, a_3, ..., a_6, a_7$, we get:

$a_2=2, a_3=3, a_4=5, a_5=6, a_6=7, a_7=11$

Substituting these values into the original equation, we get:

$log_3 (2+3+5+6+7+11)=log_3 (34)$

Therefore, the solution to the given problem is $log_3 (34)$.
 

Related to Find log_3 (a2+a3+a4+a5+a6+a7)

1. What is the equation for finding log_3 (a2+a3+a4+a5+a6+a7)?

The equation for finding log_3 (a2+a3+a4+a5+a6+a7) is log_3 (a2+a3+a4+a5+a6+a7).

2. How do I solve for log_3 (a2+a3+a4+a5+a6+a7)?

To solve for log_3 (a2+a3+a4+a5+a6+a7), you can use the properties of logarithms. First, you can rewrite the expression as log_3 (a^2+a^3+a^4+a^5+a^6+a^7). Then, you can use the product rule to simplify the expression to log_3 (a^2 * a^3 * a^4 * a^5 * a^6 * a^7). Finally, using the power rule, the answer can be written as log_3 (a^27).

3. Can I use a calculator to find log_3 (a2+a3+a4+a5+a6+a7)?

Yes, you can use a calculator to find log_3 (a2+a3+a4+a5+a6+a7). Most scientific calculators have a logarithm function that allows you to input a base and an argument to find the logarithm. Simply input the base 3 and the expression (a2+a3+a4+a5+a6+a7) to get the answer.

4. What is the value of log_3 (a2+a3+a4+a5+a6+a7) when a=1?

When a=1, the expression (a2+a3+a4+a5+a6+a7) becomes (1^2+1^3+1^4+1^5+1^6+1^7)= 1+1+1+1+1+1 = 6. Therefore, log_3 (6) is the value of log_3 (a2+a3+a4+a5+a6+a7) when a=1.

5. Is there a way to simplify log_3 (a2+a3+a4+a5+a6+a7)?

Depending on the values of a, there may be a way to simplify log_3 (a2+a3+a4+a5+a6+a7). For example, if a=3, then the expression becomes log_3 (3^2+3^3+3^4+3^5+3^6+3^7) = log_3 (3^(2+3+4+5+6+7)) = log_3 (3^27) = 27. However, in most cases, log_3 (a2+a3+a4+a5+a6+a7) is the simplest form of the expression.

Similar threads

  • General Math
Replies
1
Views
1K
Replies
4
Views
1K
Replies
8
Views
1K
  • Calculus and Beyond Homework Help
Replies
4
Views
3K
  • Introductory Physics Homework Help
Replies
23
Views
452
  • Linear and Abstract Algebra
Replies
4
Views
1K
Replies
7
Views
2K
  • Calculus and Beyond Homework Help
Replies
3
Views
1K
  • Calculus and Beyond Homework Help
Replies
2
Views
702
  • Set Theory, Logic, Probability, Statistics
Replies
9
Views
2K
Back
Top